Possible Values of J & Linearly Independent States

  • Thread starter Dishsoap
  • Start date
  • Tags
    Spin
In summary, the conversation discusses the addition of angular momenta and the construction of eigenstates for the total angular momentum. It is determined that there are 27 linearly independent states for each value of J, with J=0 as a scalar and J=1 as a vector. The Wigner 6-j symbols are mentioned as a way to add angular momenta, but in this case, since all angular momenta are j=1, there is only one way to add them. The possible values of J are not just J=3, but also include other values based on the addition of the angular momenta.
  • #1
Dishsoap
1,017
310

Homework Statement


Consider eigenstates of the total angular momentum $$\vec{J}=\vec{J}_1+\vec{J}_2+\vec{J}_3$$ where all of the [itex] \vec{J}_i [/itex] are for [itex] j=1 [/itex] , and let [itex]J(J+1)[/itex] be the eigenvalue of [itex]\vec{J}^2[/itex].

a) What are the possible values of [itex]J[/itex]? How many linearly independent states are there for each of these values?
b) Construct the [itex]J=0[/itex] state explicitly. If [itex]\vec{a},\vec{b},\vec{c}[/itex] are ordinary 3-vectors, the only scalar linear in all of them that they can form is [itex](\vec{a} \times \vec{b}) \cdot \vec{c}[/itex]. Establish the connection between this fact and your result for the [itex]J=0[/itex] state.

Homework Equations



[itex]J^2 \vert j,m \rangle=J(J+1) \vert j,m \rangle [/itex]

3. The Attempt at a Solution
[/B]
I am really struggling with this addition of angular momentum stuff, so please forgive me. Because we are more or less hinted to use [itex]J^2[/itex] instead of [itex]J_z[/itex], there is no dependence on [itex]m[/itex], so I'm thinking that for each value of J, since there are 3 values for [itex]m[/itex] because [itex]m=0,\pm{1}[/itex] there will be [itex]3^3=27[/itex] linearly independent states at least for each value of [itex]J[/itex].

I'm not quite sure how to relate [itex]J_{total}[/itex] to [itex]j[/itex]. Would I do something like

$$\langle j_1, j_2, j_3 ; m_1, m_2, m_3 \vert (J_1^2 +J_2^2+J_3^2) \vert j_1, j_2, j_3 ; m_1, m_2, m_3 \rangle = J(J+1) $$

where this then becomes

$$ j_1(j_1+1)+j_2(j_2+1)+j_3(j_3+1) = J(J+1) $$ ?

But now I'm confused, because I think that [itex]j_1=j_2=j_3=1[/itex].

I'm really just not sure how to proceed.
 
Physics news on Phys.org
  • #2
You are correct in saying that there are 27 linearly independent states.

The general way to proceed is to add two of the angular momenta to get and intermediate angular momentum and then add the third. For example you can add j1 + j2 = j12 and then j12 + j3 = J. But there is another way, j2 + j3 = j23 and then j23 + j1 = J. This kind of addition is achieved using Wigner 6-j symbols. However, your case is simpler because all angular momenta are j = 1 so there is only one way of adding them. Say you add j1 and j2 to get j12. What are the possible j12 values? Then add j3 and j12. Again what are the possible values?

As for part (b), think of J = 0 as a scalar (one thing needed to describe it, J =1 as a vector (three things needed to describe it) when the time comes to answer this part.
 
  • #3
Thank you for clarifying, but I think that I am still missing something conceptually. I do not understand why there is not only one possible value for [itex]J[/itex], namely, [itex]J=3[/itex], since we are adding three states with [itex]j=1[/itex].
 
  • #4
Dishsoap said:
Thank you for clarifying, but I think that I am still missing something conceptually. I do not understand why there is not only one possible value for ##J##, namely, ##J=3##, since we are adding three states with ##j=1##.
For the same reason that you don't get just 2 when you add two angular momenta with j = 1. Please read this http://www2.ph.ed.ac.uk/~ldeldebb/docs/QM/lect15.pdf to understand how angular momenta add.
Also, if you only got J = 3, that is 2×3 + 1 = 7 states when you know you have 27. So where are the other 20? Answer: If you account correctly for the other values alluded to in part (a), you will see.
 

Related to Possible Values of J & Linearly Independent States

1. What are possible values of J?

The possible values of J refer to the quantum numbers used to describe the angular momentum of a physical system. In quantum mechanics, J is used to represent the total angular momentum, which includes both orbital and spin angular momentum. The values of J can range from 0 to infinity, in increments of 1.

2. How are the possible values of J determined?

The possible values of J are determined by the quantum numbers associated with the system. The angular momentum quantum number, l, can take on values from 0 to n-1, where n is the principal quantum number. The spin quantum number, s, can either be 1/2 or -1/2. The total angular momentum, J, is then calculated by adding the values of l and s.

3. What are linearly independent states?

Linearly independent states refer to a set of quantum states that cannot be expressed as a linear combination of each other. In other words, the states are unique and cannot be derived from each other. In quantum mechanics, linearly independent states are important because they allow for a complete description of a physical system.

4. Why are linearly independent states important in quantum mechanics?

Linearly independent states are important in quantum mechanics because they form a basis for describing a physical system. This means that any state of the system can be expressed as a linear combination of the linearly independent states. It also allows for the calculation of probabilities and other properties of the system.

5. How are linearly independent states related to possible values of J?

The linearly independent states of a system are related to the possible values of J in that the number of linearly independent states is equal to 2J+1. This means that the values of J determine the number of unique states that can exist for a given system. For example, if J=1/2, there are 2(1/2)+1 = 2 linearly independent states.

Similar threads

  • Advanced Physics Homework Help
Replies
17
Views
1K
  • Advanced Physics Homework Help
Replies
9
Views
1K
  • Advanced Physics Homework Help
Replies
32
Views
3K
  • Quantum Physics
Replies
7
Views
1K
  • Advanced Physics Homework Help
Replies
20
Views
4K
Replies
17
Views
2K
  • Advanced Physics Homework Help
Replies
20
Views
1K
  • Advanced Physics Homework Help
Replies
6
Views
2K
  • Advanced Physics Homework Help
Replies
1
Views
1K
  • Advanced Physics Homework Help
Replies
9
Views
1K
Back
Top